Advanced Math Problem of the Week 12/20/2017

In summary, the Riemannian distance between two points in the open unit disk with the given metric is equal to the integral of the norm of the velocity vector divided by 1 minus the norm of the position vector squared. For the specific points ##-\frac{1}{2}e^{i\pi/4}## and ##\frac{1}{2}e^{i\pi/4}##, the shortest path is a straight line and the distance between them is equal to the natural logarithm of 3.
  • #1
PF PotW Robot
Here is this week's advanced math problem of the week. We have several members who will check solutions, but we also welcome the community in general to step in. We also encourage finding different methods to the solution. If one has been found, see if there is another way. Occasionally there will be prizes for extraordinary or clever methods. Spoiler tags are optional.

Consider the open unit disk ##\Bbb D\subset \Bbb C## with Riemannian metric ##ds^2 = \dfrac{\lvert dz\rvert^2}{(1 - \lvert z\rvert^2)^2}##. Find a formula for the (Riemannian) distance between two points in ##\Bbb D##, and use it to find the distance between ##-\frac{1}{2}e^{i\pi/4}## and ##\frac{1}{2}e^{i\pi/4}##

(PotW thanks to our friends at http://www.mathhelpboards.com/)
 
Physics news on Phys.org
  • #2
PF PotW Robot said:
Consider the open unit disk ##\Bbb D\subset \Bbb C## with Riemannian metric ##ds^2 = \dfrac{\lvert dz\rvert^2}{(1 - \lvert z\rvert^2)^2}##. Find a formula for the (Riemannian) distance between two points in ##\Bbb D##, and use it to find the distance between ##-\frac{1}{2}e^{i\pi/4}## and ##\frac{1}{2}e^{i\pi/4}##

(PotW thanks to our friends at http://www.mathhelpboards.com/)

Here's my attempt.

Let ##c: [a,b] → \Bbb D## be a parametrized curve in ##\Bbb D##, which is differentiable with velocity vector ##c'##.
And let ##g## be the given metric.
Then:
$$d(z_1,z_2) = \inf_{c(a)=z_1, c(b)=z_2} \int_a^b \sqrt{g(c'(t),c'(t))}\,dt
= \inf_{c(a)=z_1, c(b)=z_2} \int_a^b \frac{|c'(t)|\,dt}{1-|c(t)|^2}
$$

Due to symmetry and the global minimum of the metric at 0, the shortest path between ##-\frac 12e^{i\pi/4}## and ##\frac 12e^{i\pi/4}## is a straight line.
For that we can pick ##c(t)=te^{i\pi/4}##, so that ##|c(t)|=|t|##, ##|c'(t)| = |e^{i\pi/4}| = 1##, and:
$$\DeclareMathOperator{\artanh}{artanh}
d\left(\!-\frac 12e^{i\pi/4},\frac 12e^{i\pi/4}\!\right) = \int_{-1/2}^{1/2} \frac{dt}{1-|t|^2}
= \artanh t\bigg|_{-1/2}^{1/2} = 2\artanh \frac 12 = 2\cdot \frac 12\log\frac{1+\frac 12}{1 - \frac 12} = \log 3
$$
 
  • Like
Likes Greg Bernhardt

Related to Advanced Math Problem of the Week 12/20/2017

1. What is the topic of the Advanced Math Problem of the Week 12/20/2017?

The topic of the Advanced Math Problem of the Week 12/20/2017 is a complex mathematical problem that requires advanced knowledge and skills to solve.

2. What level of math is required to solve this problem?

This problem is designed for advanced mathematicians and typically requires a strong understanding of calculus, linear algebra, and other advanced mathematical concepts.

3. How often is the Advanced Math Problem of the Week released?

The Advanced Math Problem of the Week is released every week on Wednesday.

4. Is there a prize for solving the Advanced Math Problem of the Week?

Yes, there is a prize for solving the Advanced Math Problem of the Week. The prize varies depending on the difficulty of the problem and the number of correct solutions submitted.

5. Can I work with others to solve the Advanced Math Problem of the Week?

Yes, collaboration is allowed and encouraged for solving the Advanced Math Problem of the Week. However, only one solution per person or group will be accepted for the prize.

Similar threads

  • Math Proof Training and Practice
Replies
1
Views
1K
  • Math Proof Training and Practice
Replies
6
Views
3K
  • Math Proof Training and Practice
Replies
1
Views
2K
  • Math Proof Training and Practice
Replies
11
Views
2K
  • Math Proof Training and Practice
Replies
4
Views
1K
  • Math Proof Training and Practice
Replies
8
Views
2K
  • Math Proof Training and Practice
Replies
21
Views
2K
  • Math Proof Training and Practice
Replies
15
Views
2K
  • Math Proof Training and Practice
Replies
9
Views
2K
  • Math Proof Training and Practice
Replies
2
Views
1K
Back
Top